LSAT and Law School Admissions Forum

Get expert LSAT preparation and law school admissions advice from PowerScore Test Preparation.

 Administrator
PowerScore Staff
  • PowerScore Staff
  • Posts: 8917
  • Joined: Feb 02, 2011
|
#101102
Complete Question Explanation

Flaw in the Reasoning. The correct answer choice is (E).

Answer choice (A):

Answer choice (B):

Answer choice (C):

Answer choice (D):

Answer choice (E): This is the correct answer choice.

This explanation is still in progress. Please post any questions below!
 Jay
  • Posts: 46
  • Joined: Jan 09, 2020
|
#76311
Hello Powerscore.

May I ask why (E) is the answer?

The conclusion was "small retailers that are forced to compete with large discount chains MUST offer exceptional service in order to retain their level of profitability."

(E) says "exceptional service is not the only reason customers prefer small retail stores."

But just because the author said "MUST" doesn't mean that exceptional service is the ONLY way to retain their level of profitability.

If I say that "you must eat apple to be healthy", that doesn't mean that I'm saying eating apple is the only way to be healthy.

I don't understand why (E) is the answer..

I chose (B) instead.

(B) said "some large discount chains have lower profit margins than do some small retailers."

Is this incorrect because of the word "some"?

Thank you!
 Jeremy Press
PowerScore Staff
  • PowerScore Staff
  • Posts: 1000
  • Joined: Jun 12, 2017
|
#76416
Hi Jay,

The reason that answer choice B is incorrect is because the conclusion makes an absolute claim about what small retailers have to do retain their own (absolute) level of profitability. So the comparative possibility that answer choice B raises (that some large discount chains have lower profit margins) doesn't impact the absolute conclusion about what the small retailers themselves must do. Going a little further than that, the argument doesn't assume any specific level of profitability of large discount chains. It only starts by saying large discount chains "can" make "a" profit using their low-cost strategy. So we don't know whether the author (or the argument) assumes large discount chains have higher or lower profit margins than small retailers.

Be careful with your understanding of the conditional language in the conclusion and in the example you provided. If I say that "eating apples is necessary to be healthy," I'm saying the apples must be there and, in fact, that there is no way to be healthy that doesn't include eating apples. Now, you're right that the statement doesn't necessarily mean that there aren't other things that are necessary for health in addition to eating apples. So the statement isn't saying that eating apples is sufficient to be healthy. But it is saying that the only strategies that will work to attain health must include eating apples.

The same is true of our conclusion in the stimulus: the stimulus conclusion says that any strategy for retaining profitability MUST include offering exceptional service. The premise doesn't support that statement because all the premise says is that offering exceptional service is one (sufficient) way to maintain profitability ("if they offer exceptional service"). The possibility that answer choice E raises does undermine the conclusion, because it suggests there is another thing small retailers could do (the other thing customers prefer) that wouldn't necessarily involve offering exceptional service. Does answer choice E prove that the other reasons would maintain profitability in the absence of exceptional service? No, not exactly. But the mere possibility of such a strategy undermines the conclusion and makes it vulnerable (thus flawed).

I hope this helps!

Jeremy

Get the most out of your LSAT Prep Plus subscription.

Analyze and track your performance with our Testing and Analytics Package.